You are on page 1of 6

Chapter :- Cost Allocation, Customer Profitability Analysis, and Sales-Variance

Analysis
1) The sales-quantity variance can be decomposed into:
A) sales-mix variance and sales-volume variance
B) static-budget variance and flexible-budget variance
C) flexible-budget variance and sales-volume variance
D) market-share variance and market-size variance
Answer: D
2) The sales-volume variance is subdivided into:
A) sales-mix variance and static-budget variance
B) sales-mix variance and sales-quantity variance
C) flexible-budget variance and fixed-budget variance
D) market-share variance and static-budget variance
Answer: B

3) Flexible budget contribution margin is equal to:


A) actual contribution margin per unit times actual units sold of each product
B) actual contribution margin per unit times budgeted units sold of each product
C) budgeted contribution margin per unit times budgeted units sold of each product
D) budgeted contribution margin per unit times actual units sold of each product
Answer: D
4) Market-share variance = $390,000 (U); Market-size variance = $250,000 (F); Sales-mix variance = $660,000 (F); calculate
the sales-quantity variance.
A) $390,000 (F)
B) $640,000 (F)
C) $20,000 (F)
D) $140,000 (U)
Answer: D
Explanation: D) Sales-quantity variance = $390,000 (U) + $250,000 (F) = $140,000 (U)

5) Sales-mix variance = $310,000 (F), sales-quantity variance = $200,000 (F), flexible-budget variance = $140,000 (F),
market-size variance = $90,000 (U), calculate the sales-volume variance.
A) $740,000 (F)
B) $540,000 (F)
C) $600,000 (F)
D) $510,000 (F)
Answer: D
Explanation: D) Sales-volume variance = $310,000 (F) + $200,000 (F) = $510,000 (F)

6) Flexible-budget variance = $240,000 (F); sales-volume variance = $350,000 (U); sales-mix variance = $310,000 (F);
calculate the static-budget variance.
A) $110,000 (U)
B) $310,000 (U)
C) $310,000 (F)
D) $280,000 (F)
Answer: A
Explanation: A) Static-budget variance = $240,000 (F) + $350,000 (U) = $110,000 (U)

7) The sales-mix variance is calculated by:


A) deducting budgeted contribution margin based on actual units at budgeted mix from budgeted contribution margin

1
based on actual units sold at the actual mix
B) deducting budgeted contribution margin based on budgeted units at actual mix from budgeted contribution margin
based on actual units sold at the budgeted mix
C) deducting budgeted contribution margin based on actual units at actual mix from budgeted contribution margin based
on budgeted units sold at the budgeted mix
D) deducting budgeted contribution margin based on actual units at actual mix from budgeted contribution margin based
on actual units sold at the actual mix
Answer: A
8) The static-budget variance is the difference between:
A) an actual result and the corresponding budgeted amount in the static budget
B) the budget amount in the static budget and the amount in the flexible budget
C) an actual result and the flexible budget amount
D) the static budget amount and the sales-volume variance
Answer: A

9) More insight into the static-budget variance can be gained by subdividing it into:
A) the sales-mix variance and the sales-quantity variance
B) the market-share variance and the market-size variance
C) the flexible-budget variance and the sales-volume variance
D) the flexible-budget variance and the sales-mix variance
Answer: C

10) The static-budget variance will be favorable, when:


A) budgeted unit sales are more than actual unit sales
B) the actual contribution margin is less than the static-budget contribution margin
C) the actual sales mix shifts toward the less profitable units
D) the flexible-budget and the sales-volume variance are favorable
Answer :D
11) The sales-mix variance will be unfavorable when which of the following occurs?
A) the actual sales mix shifts toward the less profitable units
B) the contribution margin per composite unit for the actual mix is greater than the budgeted mix
C) the actual unit sales are less than the budgeted unit sales
D) the actual contribution margin is less than the static-budget contribution margin
Answer: A
12) The sales-mix variance will be favorable when:
A) the actual contribution margin is greater than the static-budget contribution margin
B) actual unit sales are more than budgeted unit sales
C) the actual sales mix shifts toward the less profitable units
D) the budgeted contribution margin for actual sales mix is greater than for the budgeted mix
Answer: D

13) The sales-quantity variance will be favorable when which of the following occurs?
A) sales-volume variance and flexible-budget variance are favorable
B) actual units of all products sold exceed budgeted units of all products sold
C) the actual sales mix shifts towards the more profitable units
D) static-budget variance and flexible-budget variance are favorable
Answer: B

14) The sales-quantity variance will be unfavorable when which of the following occurs?
A) the composite unit for the actual mix is less than for the budgeted mix
B) the actual unit sales are less than the budgeted unit sales
2
C) the actual contribution margin per unit is less than the static-budget contribution margin
D) the actual sales mix shifts toward the less profitable units
Answer: B

15) Which of the following is the formula for the sales-quantity variance?
A) deducting budgeted contribution margin based on actual units at actual mix from budgeted contribution margin based
on actual units sold at the actual mix
B) deducting budgeted contribution margin based on actual units at actual mix from budgeted contribution margin based
on actual units sold at the budgeted mix
C) deducting budgeted contribution margin based on budgeted units at actual mix from budgeted contribution margin
based on actual units sold at the budgeted mix
D) deducting budgeted contribution margin based on budgeted units at budgeted mix from budgeted contribution
margin based on actual units sold at the budgeted mix
Answer: D

16) The sales-quantity variance results from a difference between:


A) the actual sales mix and the budgeted sales mix
B) the actual quantity of units sold and the budgeted quantity of unit sales in the static budget
C) actual contribution margin and the budgeted contribution margin
D) actual market size in units and the budgeted market size in units
Answer: B
17. The following information is provided for the month of March:

Actual Budget
Calamine Capity Calamine Capity
Sales in pounds 3,840 lbs. 3,980 lbs. 4,400 lbs. 3,300 lbs
Price per pound $3.00 $3.00 $2.00 $3.00
Variable cost per pound 1.10 2.20 1.00 1.50
Contribution margin $1.90 $0.80 $1.00 $1.50

Budgeted and actual fixed corporate-sustaining costs are $1,850 and $2,300, respectively. What is the contribution margin
for the flexible budget?
A) $10,480
B) $9,810
C) $9,596
D) $12,330
Answer: B
Explanation: B) Contribution margin for the flexible budget = (3,840 × $1) + (3,980 × $1.50) = $9,810

18.The following information is provided for the month of March:

Actual Budget
Calamine Capity Calamine Capity
Sales in pounds 3,800 lbs. 3,990 lbs. 4,500 lbs. 3,500 lbs
Price per pound $2.80 $2.80 $2.00 $3.00
Variable cost per pound 1.00 2.00 1.00 1.50
Contribution margin $1.80 $0.80 $1.00 $1.50

Budgeted and actual fixed corporate-sustaining costs are $1,850 and $2,300, respectively.

3
For the contribution margin, what is the total static-budget variance?

A) $6,840 favorable
B) $282 unfavorable
C) $35 favorable
D) $282 favorable
Answer: D
Explanation: D) Total static-budget variance
= [(4,500 × $1.00) + (3,500 × $1.50)] - [(3,800 × $1.80) + (3,990 × $0.80)]
= $9,750 - $10,032 = $282 favorable

19) The Corata Appliance Manufacturing Corporation manufactures two vacuum cleaners, the Standard and the
Super. The following information was gathered about the two products:

Standard Super
Budgeted sales in units 2,700 900
Budgeted selling price $600 $1,700
Budgeted contribution margin per unit $400 $1,050
Actual sales in units 3,200 1,500
Actual selling price $650 $1,680

What is the budgeted sales-mix percentage for the Standard and the Super vacuum cleaners, respectively?
A) 0.75 and 0.25
B) 0.68 and 0.32
C) 0.25 and 0.75
D) 0.32 and 0.68
Answer: A
Explanation: A)
Budgeted sales-mix percentage for the Standard = 2,700 / (2,700 + 900) = 0.75
Budgeted sales-mix percentage for the Super = 900 / (2,700 + 900) = 0.25

20.The following information was gathered about the two products:

Standard Super
Budgeted sales in units 2,700 600
Budgeted selling price $600 $1,700
Budgeted contribution margin per unit $420 $1,060
Actual sales in units 2,800 1,200
Actual selling price $650 $1,680

What is the total sales-volume variance in terms of the contribution margin?


A) $678,000 unfavorable
B) $594,000 favorable
C) $636,000 unfavorable
D) $678,000 favorable
Answer: D
Explanation: D)
Standard = (2,800 - 2,700) × $420 = $42,000 F
Super = (1,200 - 600) × $1,060 = 636,000 F
$678,000 F
4
21) Archoid's Flowering Plants provides the following information for the month of May:

Actual Budget
Tulips Geraniums Tulips Geraniums
Sales in units 4,600 4,400 4,950 3,300
Contribution margin per unit $13 $19 $10 $22

What is the budgeted contribution margin per composite unit for the budgeted mix? (Round any intermediary
calculations two decimal places.)
A) $16.12
B) $14.80
C) $19.00
D) $15.88
Answer: B
Explanation: B9) Budgeted contribution margin per composite unit for the budgeted mix
=
(10x4950)+(3300x22) =14.8$

4950+3300

21) The Corata Appliance Manufacturing Corporation manufactures two vacuum cleaners, the Standard and the
Super. The following information was gathered about the two products:

Standard Super
Budgeted sales in units 2,400 700 total BQ3100
Budgeted selling price $600 $1,700
Budgeted contribution margin per unit $400 $1,070
Actual sales in units 3,000 1,300 total AQ4300
Actual selling price $650 $1,680

22.What is the total sales-quantity variance in terms of the contribution margin? (Round intermediary calculations to
two decimal places.)
A) $369,600 favorable
B) $295,320 favorable
C) $664,920 favorable
D) $74,280 favorable
Answer: C
Explanation: C)
Sales mix calculations:
Standard = 2,400 / (2,4070 + 700) = 0.77
Super = 700 / (2,400 + 700) = 0.23

Sales quantity variance:


Standard = (4,300 - 3,100) (total AQ-total BQ) × 0.77 × $400 = $369,600 F AQ more than BQ
Super = (4,300 - 3,100) × 0.23 × $1,070 =295,320 F
$664,920 F

23.. The following information was gathered about the two products:

Standard Super
Budgeted sales in units 2,700 900 total BQ3600
Budgeted selling price $600 $1,700
5
Budgeted contribution margin per unit $700 $1,050
Actual sales in units 3,000 1,200 total AQ 4200
Actual selling price $650 $1,680

What is the total sales-mix variance in terms of the contribution margin? (Round intermediary calculations to two
decimal places.)
A) $117,600 favorable
B) $58,800 favorable
C) $294,000 favorable
D) $176,400 favorable
Answer: B
Explanation: B)
Sales mix calculations:
Budgeted Standard = 2,700 / (2,700 + 900) = 0.75
Budgeted Super = 900 / (2,700 + 900) = 0.25
Actual Standard = 3,000 / (3,000 + 1,200) = 0.71
Actual Super = 1,200 / (3,000 + 1,200) = 0.29

Sales mix variance: total AQ*(A sales mix -B sales mix )* B CM


Standard = 4,200 × (0.71 - 0.75) × $700 = $117,600 U
Super = 4,200 × (0.29 - 0.25) × $1,050 = $176,400 F
$58,800 F
24) The market-share variance is the difference in budgeted contribution margin for actual market size in units caused
solely by actual market share being different from budgeted market share.
Answer: TRUE
25) sales-quantity variance occurs in a multi-product company from a change in the sales mix.
Answer: FALSE.
26)

You might also like